Si un conjunto es cerrado bajo uniones e intersecciones, ¿es cerrado bajo complementos?

Estoy pensando en álgebras sigma aquí, que son conjuntos (no vacíos) cerrados bajo uniones contables, intersecciones contables y complementos.

Pero solo necesitas 2 de estas condiciones para garantizar la tercera:

  1. Si un conjunto (no vacío) se cierra bajo uniones contables y complementos, entonces se cierra bajo intersecciones contables (contable De Morgan).

  2. Si un conjunto (no vacío) se cierra bajo intersecciones y complementos contables, entonces se cierra bajo uniones contables (contable De Morgan).

Ahora pregunto:

  • Si un conjunto (no vacío) X es cerrado bajo uniones contables e intersecciones contables, entonces es cerrado bajo complementos? (¿Cambia esto si X es finito, contable o incontable?)

(Digo "conjunto" porque en ZFC todo es un conjunto, pero la gente a menudo los llama "familias" o "colecciones", es decir, conjuntos de conjuntos).

Preguntas extra:

  • Si un conjunto es cerrado bajo uniones finitas, ¿entonces es cerrado bajo uniones contables?
  • Si un conjunto es cerrado bajo intersecciones finitas, ¿entonces es cerrado bajo intersecciones numerables?
Las respuestas son No para los tres.

Respuestas (2)

Digamos X es una familia de subconjuntos de Ω .

Un problema de tener uniones e intersecciones pero no complementos es que tal vez haya un elemento X Ω que no se encuentra en ningún conjunto de X . Ninguna cantidad de uniones e intersecciones te conseguirá X , pero todo complemento contendrá X .

Incluso si X = Ω , tenemos problemas. Por ejemplo, deja Ω = R y deja X consisten en todos los intervalos de la forma ( , a ) y ( , a ] . Esto está cerrado bajo uniones e intersecciones, por lo que nunca obtendremos un intervalo que sea infinito en la otra dirección.

Una construcción similar funciona para subconjuntos del conjunto finito. { 1 , 2 , , norte } o el conjunto numerable infinito norte , también.


La mayoría de las topologías son ejemplos de por qué las respuestas a sus preguntas adicionales son negativas. Si X es el conjunto de todos los conjuntos abiertos en R (con respecto a la topología usual) entonces X está cerrado bajo uniones arbitrarias e intersecciones finitas, pero no bajo intersecciones infinitas. Por ejemplo, norte = 1 ( 1 norte , 1 norte ) = { 0 } , que no está abierto.

De manera similar, el conjunto de todos los conjuntos cerrados tendrá uniones finitas e intersecciones arbitrarias, pero no uniones infinitas.

Qué respuesta tan amable y legible, gracias.

Para cualquier X , elegir Y X , entonces PAG ( Y ) se cierra bajo intersección/unión arbitraria, pero no complemento. En la teoría de la medida, la diferencia de conjuntos X Y se usa más a menudo que el complemento. Elija una secuencia creciente de conjuntos X 1 X 2 X 3 , entonces { X 1 , , X 2 , } { i X i } es cerrado bajo una unión e intersección arbitrarias, pero no un conjunto de diferencias.

Todos los conjuntos finitos de un conjunto infinito X está cerrado bajo uniones finitas, pero no uniones contables.

Todos los conjuntos cofinitos de un conjunto infinito X se cierra bajo intersecciones finitas, pero no numerables.